Calculating Cone Surface Area | Step-by-Step Guide

Click For Summary
SUMMARY

The discussion focuses on calculating the surface area of a cone using integral calculus. The key point addressed is the confusion surrounding the appearance of the -1 in step 4 of the calculation. The integral is defined as \(\int^{b}_{a}f(x)dx\), which simplifies to \(F(b) - F(a)\). The -1 arises from substituting the lower limit (x = 0) into the integral, which is then subtracted from the upper limit (x = 1/2).

PREREQUISITES
  • Understanding of integral calculus
  • Familiarity with the Fundamental Theorem of Calculus
  • Knowledge of function notation and limits
  • Basic skills in evaluating definite integrals
NEXT STEPS
  • Study the Fundamental Theorem of Calculus in detail
  • Practice evaluating definite integrals with varying limits
  • Explore applications of integral calculus in geometry, specifically cone surface area
  • Learn about common pitfalls in integral calculations and how to avoid them
USEFUL FOR

Students studying calculus, educators teaching integral calculus, and anyone looking to deepen their understanding of geometric applications of integrals.

bobsmith76
Messages
336
Reaction score
0

Homework Statement

Screenshot2012-02-05at43959AM.png


I understand everything except for the part where the -1 pops out of nowhere on step 4. why? how?
 
Physics news on Phys.org
Remember that when you solve an integral, you need to take the integral of the highest value minus lowest value. So if you want to solve

\int^{b}_{a}f(x)dx

Then you'll have

=\left[F(x)\right]^b_a

=F(b)-F(a)
 
the 1 is the result of substituting the lower limit of the previous step, ie x = 0, which subtracted from the upper limite of the previous step, x = 1/2

edit: oops. too late
 
thanks, I got it.
 
Question: A clock's minute hand has length 4 and its hour hand has length 3. What is the distance between the tips at the moment when it is increasing most rapidly?(Putnam Exam Question) Answer: Making assumption that both the hands moves at constant angular velocities, the answer is ## \sqrt{7} .## But don't you think this assumption is somewhat doubtful and wrong?

Similar threads

  • · Replies 7 ·
Replies
7
Views
2K
  • · Replies 4 ·
Replies
4
Views
2K
  • · Replies 7 ·
Replies
7
Views
2K
Replies
3
Views
2K
  • · Replies 33 ·
2
Replies
33
Views
3K
Replies
11
Views
3K
  • · Replies 3 ·
Replies
3
Views
2K
  • · Replies 10 ·
Replies
10
Views
2K
  • · Replies 2 ·
Replies
2
Views
2K
  • · Replies 4 ·
Replies
4
Views
2K